You are on page 1of 47

A 40-year-old man presents with recurrent non-productive cough, headache, and dyspnea,

and sometimes chills. These seems to occur late in the day on most days. He is a non-smoker
and works cleaning cages at a local zoo, but says that his symptoms don’t occur every day and
sometimes occur after he leaves work. He has lost about 6 pounds in weight over the last 2
months. PFTs show mild restriction. A chest x-ray shows mid-upper lung interstitial infiltrates.

a) sarcoidosis
b) idiopathic BOOP (bronchiolitis obliterans with organizing pneumonia)
c) interstitial fibrosis related to collagen-vascular disease
d) idiopathic pulmonary fibrosis (IPF)
e) hypersensitivity pneumonitis

The correct answer is choice E.

There are some key clues in the history presented here that will point us towards the most
likely diagnosis.

the symptoms are recurrent

they occur late in the day or after work

the occupational history is suggestive of allergen exposure

there is a history of weight loss

there is a restrictive pulmonary function defect

there are xray changes

there are no multisystem features

These features as a group are not compatible with the multisystem illness sarcoidosis. The
clear recurrent nature and consistent timing of the symptoms, and the absence of symptoms
certain days, goes against a progressive interstitial disease such as IPF, or interstitial fibrosis
from a collagen disease, or idiopathic BOOP, which classically presents with a flu-like illness
from which the patient fails to recover to baseline, and develops progressive symptoms.

The only logical answer to this question is hypersensitivity pneumonitis, previously known as
extrinsic allergic alveolitis.

Hypersensitivity pneumonitis represents a respiratory illness resulting from exposure to


inhaled allergens to which a patient has been previously sensitized. It presents with flu-like
symptoms, cough, and shortness of breath that are temporally related to allergen exposure.
There is classically a restrictive defect on PFTs with reduced carbon monoxide transfer.The
time between heavy exposure and symptoms is often 4-8 hours, consistent with the time
scale of T-cell activation, so symptoms often occur after occupation exposure is over. Patients
may be symptom free on days when exposure is limited or absent. There is significant
inflammation within the lungs, contributing to radiographic infiltrates and weight loss.

The list of potential triggers is very large. Classically the illness results from exposure to
mouldy hay or farm dust (Farmer's lung), or bird feathers and excrement (Pigeon-fancier's
lung). However, many industrial substances and dusts have been implicated, as well as
micro-organisms present within the materials being worked with. Thus we have
woodworker's lung, sauna taker's lung, animal handler's lung etc. The patient here will have
been exposed to animal dander and excrement repeatedly, and the inhalational load will have
been substantial during brushing and shovelling activities. The time-scale from exposure to
symptoms is classical, as is the absence of symptoms on certain days.

Hypersensitivity pneumonitis occurs more frequently in non-smokers. The acute form


demonstrated by this patient is associated with immune complex deposition in the lungs.
Resolution of the acute form can take days after exposure ceases, and careful history taking
may be required to identify the precipitant, taking into account the time delay in symptoms.

Diagnosis is not usually difficult provided a clear history is obtained, searching for exposure,
periodicity, and symptom free periods. Without the history the clinical features can be
consistent with other interstitial lung diseases. Diagnosis relies on the presence of a number
of features:

exposure to a known allergen

recurrent episodes of symptoms

symptoms occurring 4-8 hours after exposure

weight loss

inspiratory crackles on chest exam

development of antibody to that allergen

Another predictive tool was developed by Girard et al (see references). If the first five
elements above are present, the probability of hypersensitivity pneumonitis is 98% if
precipitating antibodies are present, and 92% if they are not. If only the first four elements
are present then the probability is 93% if antibodies are present and 72% if not.

Failure to make the diagnosis is associated with ongoing exposure, which can alter the
disease from an acute intermittent form to a chronic form more likely to be associated with
the development of emphysematous change or lung fibrosis, and with poorer outcome.
Treatment relies on avoiding the trigger, and on the administration of glucocorticoids.

You are called to assess a 4600g newborn who was noted be cyanotic. Upon arrival, you note
a bluish discoloration of the patient's extremities, face, and trunk. An initial arterial blood gas
analysis indicates a pAO2 of 40 mm Hg. Physical examination reveals a grade 2/6 systolic
murmur and a loud second heart sound.

The chest radiograph reveals a normal sized heart and decreased pulmonary vascular
markings. After placing the infant under a hood containing 100% oxygen, his cyanosis
improves, and you obtain the following arterial blood gas sample:
pH = 7.36
pCO2 = 37 mm Hg
pAO2 = 101 mm Hg
HCO3- = 20
Base excess = -3

Which of the following is the most appropriate interpretation of this infant's diagnostic
evaluation?

a) The normal pAO2 on 100% oxygen indicates that cyanosis is likely due to
methemoglobinemia

b) The arterial blood gas suggests alveolar hypoventilation is the mechanism for cyanosis

c) There is increased pulmonary vascular resistance, leading to a right-to-left shunt at the


atrial or ductal levels

d) There is a right-to-left shunt at the ventricular level, likely due to complex congenital heart
disease

e) This is acrocyanosis, and is a normal finding in a young newborn

The correct answer is choice C

This infant has persistent pulmonary hypertension of the newborn (PPHN). Remember that in
fetal life, oxygenation occurs in the placenta - the resistance in the pulmonary vascular bed is
high, and blood is shunted away from the fetal lungs through the ductus arteriosus and the
foramen ovale. At birth, however, the pulmonary vascular resistance should drop, beginning
with the infant's first breath. This allows blood to enter the lungs to be oxygenated, and the
foramen ovale and ductus arteriosus should close within hours to days of birth.

In some infants, this process does not occur - the vascular resistance in the pulmonary bed
remains high. Because blood follows the path of least resistance, it is thus "easier" for the
blood to pass through the ductus arteriosus or foramen ovale and enter the systemic
circulation, without ever passing through the lungs to be oxygenated. This causes hypoxemia
and cyanosis.

PPHN can result from a variety of conditions, most notably meconium aspiration syndrome
and congenital diaphragmatic hernia. Infants of diabetic mothers also have an increased rate
of PPHN - and this infant's large size suggests that may be the case in this vignette.

Even if you didn't recognize that this was PPHN, the clues in the stem should have led you to
that physiologic explanation. For example, the chest x-ray shows decreased pulmonary
vascular markings, indicating that blood is not ever entering the lungs. Also, the single, loud
S2 is a marker of pulmonary hypertension.

Methemoglobinemia (choice A) can cause cyanosis, but the pAO2 should improve markedly
with the administration of 100% oxygen. Classically, patients with methemoglobinemia will
remain cyanotic even on 100% O2, even though their pAO2 is high.
Hypoventilation (choice B ) can cause hypoxemia and cyanosis, but would also cause CO2
retention.

Complex congenital heart disease (choice D) could cause cyanosis. However, the
improvement that this patient experienced with 100% oxygen suggests that this is not the
case. Even with 100% oxygen, infants with cyanotic heart disease will seldom attain a pO2 of
greater than 100 mm Hg, or have a rise of greater than 10-30 mm Hg from their
pre-hyperoxygenated baseline. Also, in this case, there are clues in the chest x-ray and
physical exam that suggest an alternate mechanism of hypoxemia.

Acrocyanosis (choice E) is a bluish discoloration of the extremities that is a normal finding in


infants. It is not associated with systemic hypoxemia. This infant's cyanosis is initially severe
and pronounced, and associated with a markedly decreased pAO2.

A 13 year old girl with systemic lupus erythematosus complains of low-grade fever and chest
pain that started today. On examination, you note a heart rate of 120 bpm and a temperature
of 38.3 C. She ill appearing, and prefers to sit, leaning forward, instead of lie down. You are
concerned about pericarditis and decide to admit her for further investigations and
treatment.

Which of the following additional findings would be LEAST likely in this clinical situation?

a) A rumbling, mid-diastolic murmur heard best at the apex


b) A drop in systolic blood pressure of >10mm Hg with inspiration
c) A harsh, rasping, to-and-fro sound on cardiac auscultation
d) Worsening of pain with deep inspiration
e) An electrocardiogram (ECG) showing diffuse ST wave elevation

The correct answer is choice A.

Pericarditis is a commonly-tested illness on medical exams, due both to its clinical importance
and its abundance of characteristic clinical features. The latter are worth committing to
memory and are described below. (An additional characteristic feature - that patients often
refuse to lie down, and may assume a characteristic posture of sitting forward, looking
anxious - is described in the vignette.)

Pericarditis is most commonly caused by viral infections, but connective tissue disorders (such
as SLE or rheumatoid arthritis) are frequently associated with pericarditis, as is recent cardiac
surgery, uremia, and tuberculosis.

A mid-diastolic, rumbling murmur heard best at the apex (choice A) is an Austin-Flint murmur,
and is a sign of aortic regurgitation, not pericarditis. (However, this sign is commonly tested in
its own right!)

A drop of >10mm Hg on inspiration (choice B ) is pulsus paradoxus, a ominous sign of


increased pressure around the heart that can occur in pericarditis, cardiac tamponade,
tension pneumothorax, or severe croup. It is normal for the blood pressure to drop (and
heart rate to increase) with deep inspiration - this occurs because the decreased intrathoracic
pressure during inspiration allows blood to pool in the lungs, decreasing the amount that
returns to the left ventricle and transiently decreasing cardiac output. However, when the
heart is constricted, as in pericarditis, this phenomenon is exaggerated, as the pressure from
a constrictive pericardium leads to an additional decrease in cardiac output.

A harsh, to-and-fro sound on cardiac auscultation (choice C) is a friction rub - a cardinal sign
of pericarditis. This sign may also be described as having a squeaky, grating, or scratching
character; it consists of three discrete components, two in diastole and one in systole.

Worsening of chest pain on inspiration (choice D) is described as pleurisy or pleuritic chest


pain. It is most commonly caused by diseases that irritate the pleura, which occurs during
inspiration. However, it frequently occurs in pericarditis, for the same reasons that a pulsus
paradoxus occurs.

ST segment elevation (choice E) can indicate myocardial ischemia, especially when it occurs in
a vascular distribution. However, diffuse ST elevation, especially upwardly concave elevation,
may indicate pericarditis, and is a very commonly-tested on standardized medical exams.

A 60-year-old man complains of left eye pain and photophobia. On examination there is an
erythematous rash of the periorbital skin and forehead on the left side. You also note
erythema and a small vesicle on the tip of the patient’s nose. He is otherwise healthy and is
currently being treated for hypertension and dyslipidemia. He has no current drug allergies.

What is the most appropriate therapy for this patient?

a) Oral acyclovir 800mg three times a day for 7-10 days


b) Oral valacyclovir 500 mg three times a day for 7 days
c) Oral famciclovir 500mg three times a day for 7 days
d) Oral prednisolone, 40mg daily for 7 days
e) Topical acyclovir, 5 times a day until resolution of ocular symptoms

The correct answer is choice C

This patient has herpes zoster ophthalmicus, a condition caused by a reactivation of a latent
infection with the varicella-zoster virus (VZV). The initial infection is a benign febrile illlness
(chickenpox). Following this infection, the VZV can remain dormant in the dorsal root ganglion
for many years until a decline in the patient's immune response due to aging, stress, poor
nutrition, or immuno-suppressive illness or therapy allows a reactivation of the virus
producing herpes zoster (shingles). In herpes zoster ophthalmicus, the virus is released from
the trigeminal ganglion, travels down the ophthalmic division of the nerve to the nasociliary
nerve where the nerve divides to innervate the surface of the globe and the skin of the nose
down to its tip. Manifestations include a localized vesicular rash along the dermatomal
distribution of the nerve which respects the midline, dermatomal pain, conjunctivitis,
episcleritis and scleritis, decreased corneal sensation, a punctate keratitis which can develop
into a dendritic keratitis that stains with fluorescein or rose bengal, and uveitis which can
cause photophobia.

The mainstay of treatment for herpes zoster ophthalmicus are oral antiviral agents. Oral
acyclovir, 800mg 5 times a day for 7-10 days, has been shown to reduce the duration and
severity of symptoms as well as reduce the incidence and severity of complications. Other
antiviral agents which have been shown to be equally safe and effective for herpes zoster
ophthalmicus are valacyclovir and famciclovir, whose more favorable pharmacokinetic
profiles and simpler dosing regimens make them the current preferred antiviral over acyclovir.
Valacyclovir is given at 1000 mg 3 times a day for 7 days or Famciclovir, is given at 500mg 3
times a day for 7 days.

Some studies have shown that the use of oral steroids, choice D, when used in combination
with an antiviral, provided some benefit in reducing the incidence and severity of acute pain
but showed little benefit in preventing post-herpetic neuralgia. Though corticosteroids may
be used in patients over the age of 50, they should be used with caution in patients with
comorbid conditions such as diabetes or gastritis, and only in conjunction with an oral
antiviral.

Systemic antiviral therapy has largely replaced topical antiviral preparations, choice E, for
treating ocular complications of herpes zoster ophthalmicus and should not be used routinely
without consultation with an ophthalmologist.

A 2 year old girl with unrepaired tetralogy of Fallot presents to the emergency department
with increased cyanosis. Physical examination shows a small, very cyanotic child lying on a
stretcher with her knees drawn up against her chest. Pulse oximetry shows an oxygen
saturation of 58% in room air. Her respirations are rapid and deep. Auscultation of the heart
discloses tachycardia but no murmur.

Which of the following is the most appropriate general treatment strategy for this patient?
a) Decrease systemic vascular resistance to increase right-to-left shunting
b) Decrease pulmonary vascular resistance to decrease left-to-right shunting
c) Decrease systemic vascular resistance to decrease right-to-left shunting
d) Increase pulmonary vascular resistance to increase left-to-right shunting
e) Increase systemic vascular resistance to decrease right-to-left shunting

The correct answer is choice E.

Understanding the anatomy of patients with congenital heart disease is crucial for
understanding their care. Often, the best way to think about the anatomy for a patient with
congenital heart disease is to imagine the path of blood flow. For example, imagine a single
red blood cell in a normal patient. The red blood cell returns to the heart via the vena cava,
deoxygenated. It then enters the right atrium, then the right ventricle, then goes to the lungs
via the pulmonary artery. After getting oxygenated in the pulmonary capillaries, it returns to
the left atrium via a pulmonary vein, then enters the left ventricle, then moves out to the
body to deliver its oxygen to the tissues.

Patients with Tetralogy of Fallot have four distinct lesions ("PROVe": pulmonary outflow tract
obstruction, right ventricular hypertrophy, an overriding aorta, and a large VSD) but only two
of them are necessary for understanding the pathophysiology of the lesion: the pulmonary
outflow tract obstruction and the VSD.

Imagine now a single red blood cell returning to the heart in a patient with Tetralogy of Fallot.
After leaving the vena cava and the right atrium, the RBC enters the right ventricle. Here, the
cell must make a "choice" - to pass through the stenotic pulmonary valve and enter the lungs,
or to pass through the VSD and into the left ventricle (and on to the systemic circulation)
without ever getting oxygenated? Remember, blood flows to the path of least resistance, and
thus how much blood goes to the lungs versus how much goes to the body without being
oxygenated depends on the balance between the pulmonary vascular resistance and the
systemic vascular resistance.

When the pulmonary vascular resistance is high and the systemic vascular resistance is low, it
is much more difficult for blood to get into the lungs than it is for it to pass through the VSD
and out to the circulation without being oxygenated. This leads to severe right-to-left
shunting, called a "Tet spell," and is what is happening to the patient in the vignette.

Alternately, if the systemic vascular resistance is high and the pulmonary vascular resistance
is low, it is easier for blood to pass through the pulmonary outflow tract obstruction and into
the lungs, where it can be oxygenated. This is the goal of treatment for a Tet spell. Thus, the
correct answer is choice E - if the systemic vascular resistance increases, then the right-to-left
shunt should decrease - which will reduce the patient's cyanosis.

Decreasing the systemic vascular resistance would lead to increased right-to-left shunting
(choice A), but this will cause more cyanosis and would not be the goal of treatment.

Decreasing pulmonary vascular resistance (as in choice B ) is a good idea, as it would allow
more blood to enter the lungs - but the goal is to decrease the right-to-left shunt, not the
left-to-right.

Decreasing systemic vascular resistance (choice C) will lead to more right-to-left shunting,
which is not what this patient needs.
Increasing pulmonary vascular resistance (choice D) would cause increased right-to-left
shunting, which as described previously, would be very bad in this situation.

You are looking after a 50 year old man who has severe acute pancreatitis secondary to biliary
tract stones. He develops chills and rigors, and spikes a fever of 39.5 Celsius. He is
complaining of increasing abdominal pain.

On exam he is jaundiced, and his blood pressure is 100/70 mmHg when supine falling to
90/60 mmHg when sitting, with an increase in heart rate from 95 to 120 beats per minute. His
jugular venous pressure appears to be low. His abdomen is distended and clinically consistent
with ascites. Over the next few hours his urine output falls from 50 ml/h to 15 ml/h.

Which of the following statements is FALSE?

a) This man is at high risk of acute kidney injury due to obstructive jaundice and hypovolemia

b) Urine sodium of 10 mmol/L would be consistent with ischemic ATN

c) He should receive boluses of isotonic saline or colloid until the JVP and BP are normalized,
with frequent review of the physical findings

d) At this time administration of furosemide to increase urine flow is contraindicated

e) Renal function should be monitored at least daily using serum creatinine

The correct answer is choice B.

The classical teaching is that a urinary sodium of < 20 mmol/L is supportive of a diagnosis of
hypovolemia or pre-renal oliguria, whereas ATN is classically associated with a urinary sodium
of > 40 mmol/L. The logic behind this is that in hypovolemia, the physiological response in
defense of intravascular volume is the retention of sodium and water, a process encouraged
by activation of the renin-angiotensin-aldosterone system. In ATN, tubular damage means
that sodium reabsorption is incomplete, and sodium spills out into the urine. There are some
problems with this approach however.

bilateral renal artery stenosis (whole kidney ischemia) or severe glomerulonephritis


(glomerular ischemia) can lead to an artificially low renal sodium even in normovolemia i.e.
the kidneys behave as if they are pre-renally impaired

diuretics can lead to inappropriately high urinary sodium despite volume depletion
many patients have intermediate urinary sodium levels (20-40 mmol/L)

One way around the limitations of urinary sodium is to calculate the fractional excretion of
sodium (FENa). This is the quantity of sodium excreted divided by the quantity filtered, and
expressed as a percentage. The equation for this calculation is (UNa x PCr) / (PNa x UCr) with
the result multiplied by 100 to give the FENa as percentage. In hypovolemia, sodium
reabsorption is appropriately enhanced and the FENa is < 1%. When tubular injury occurs
(ATN) the FENa rises to > 2%.

Obstructive jaundice and hypovolemia do indeed place this man at high risk of acute kidney
injury. In severe pancreatitis there is extensive third space fluid loss, and patients often
require large volumes of intravenous fluid replacement to maintain their cardiac preload.
These should be administered to clinical euvolemia, at least initially, using end-points such as
BP (including postural change), heart rate, and urine output. Central venous pressure
measurement can be useful in some patients, but it is the trend in the CVP and the response
to fluid that is important, not just the number. The controversy over whether crystalloids or
colloids should be used is still ongoing.

Proponents of colloids point to the inevitable redistribution of crystalloid to the extravascular


space after a very short time, leading to very significant edema both visible in the peripheries
and flanks, and invisible in internal organs. Crystalloid proponents point to the lack of studies
demonstrating better outcome, and to the additional expense of colloid. Regardless of the
fluid used, clinical endpoints must be decided upon, targeted, and frequently reassessed, as
should renal function (creatinine).

The administration of furosemide to a hypovolemic oliguric patient, simply to increase their


urine output, is without physiological basis or clinical justification. There are a number of
events during acute pancreatitis that contribute to renal impairment, and none is corrected
by furosemide.

Hypovolemia due to third space fluid loss

peripheral vasodilatation due to the systemic inflammatory response syndrome (SIRS) - this is
the reason for the early rise in CRP and WBC in pancreatitis, not necessarily infection
(although this should of course be excluded)

increased intra-abdominal pressure - it is critical that this not be forgotten. Intra-abdominal


hypertension or the more advanced intra-abdominal compartment syndrome result in renal
venous congestion, potential ureteric obstruction, and a reduction in renal perfusion pressure.
If this is not relieved (through drainage of free intra-abdominal fluid or decompressive
laparotomy) multiple organ failure will follow as gut and renal perfusion continue to fall.
Intra-abdominal pressure can be measured using a pressure transducer attached to the
sampling port on the urinary catheter, which is then occluded after 25 ml of normal saline is
injected in to the bladder.
A 50 year old man has surgery for diverticulosis. Two days post-op he becomes hypotensive
and febrile to 103F. A blood culture is positive for gram negative rods. The next morning, he is
noted to be tachypneic and agitated. A chest x-ray shows bilateral alveolar infiltrates. Right
heart (Swan-Ganz) catheterization reveals a pulmonary capillary wedge pressure of 9 mm Hg
(normal 5-10). His blood gas breathing room air shows a PaO2 of 50 mm Hg and a PaCO2 of
27 mm Hg.

What is his A-a gradient (AaDO2, PAO2-PaO2)?


(Assume barometric pressure is 760 mm Hg, water vapor pressure is 47 mm Hg, and RQ is
0.8)

A) - < 10
B) - 10 – 25
C) - 26 – 50
D) - 51 – 75
E) - > 75

The correct answer is choice D.

The Alveolar-arterial (A-a) gradient is calculated by the formula:

PA02-Pa02

Where PA02 = Fi02 x (pAtm -pH20) - PaCo2/RQ

Fi02 is the fraction of inspired oxgyen, 21% for a patient breathing room air.

pAtm is atmospheric or barometric pressure, usually assumed to be 760 mm Hg or Torr, the


average pressure at sea level.

pH20 is water vapor pressure, 47 mm Hg at 37oC.

RQ is the respiratory quotient or the ratio of carbon dioxide produced to oxygen consumed by
the body under steady-state conditions. It varies between 0.7 to 1 with an average of 0.8.

Using this formula and the values specified the patient has an A-a gradient of 66, therefore
answer D is correct.

The A-a gradient is a measure of the efficiency of blood oxygenation. It is altered by


ventilation/perfusion defects or diffusion problems but not by hyper- or hypoventilation.
Normal values vary with age but generally are between 7 - 14 for a patient breathing room air.

This patient's elevated A-a gradient is most likely the result of a diffusion defect or
ventilation/perfusion mismatch, also called a shunt. That is, the lungs are perfused normally
but, due to his apparent pneumonia, the alveoli are filled with fluid and unable to be
adequately ventilated.
A 23-year-old man with rheumatic mitral regurgitation presents to your clinic with frequent
palpitations and exercise intolerance.
He is currently in no distress and other wise appears healthy. He is a current smoker and has a
10 pack-year smoking history. He has no history of diabetes and has never been hospitalized
since he had rheumatic fever as a child.

Which one of the following is NOT an indicator of the severity of his mitral regurgitation?

a Third heart sound


b The loudness of the murmur
c Signs of heart failure
d Presence of thrill
e S4

The correct answer is choice E.


Mitral valve regurgitation occurs when there is valvular insufficiency and blood regurgitates
from the left ventricle into the left atrium. The most common cause is mitral valve prolapse,
rheumatic heart disease is becoming less common.

A third heart sound (Choice A) occurs when the left ventricle becomes dilated or fails and is
related to rapid filling. It therefore is common in more severe cases of mitral valve
regurgitation. A "gallop" rythm can be heard in patients with tachycardia and left ventricular
failure.

The loudness of the murmur (Choice B) is proportionate to the degree of regurgitation.

Signs of heart failure (Choice C) occur when mitral valve regurgitation is more advanced and
often with long standing cases.

The presence of a thrill (Choice D) is related to the loudness of the murmer and therefore the
extent of mitral valve regurgitation.

The fourth heart sound (Choice E) is associated with increased stiffness of the ventricle and
causes include: hypertrophy, overloading or fibrosis of the ventricle, long-standing
hypertension, aortic stenosis and Congestive Heart Failure. It is not associated with mitral
regurgitation.
A 35 year-old female presents at the ER with anxiety, fatigue, dizziness, nausea and vomiting,
reporting a loss of 10 kg during the last year. She is thin, with a BMI of 16. She has notes
increased generalized skin pigmentation over the last year and a half. Counseling that she
received for he feelings of anxiety during the last three months has not helped any of her
symptoms. Her supine blood pressure is 94/58 mmHg and standing 81/55 mmHg, so you note
slight orthostatic intolerance. Her pulse is 82/min. Her sodium level is 119 mmol/l (normal
135-145), potassium is 5.3 mmol/l (normal 3.5-5.0). Cortisol is extremely low even when
challenged with ACTH, and ACTH is very high. On further testing, she tests positive for adrenal
cortex antibodies.

Management of this patient should include all of the following except:


A ) IV hydrocortisone
B ) thyroid function tests
C ) IV hypertonic sodium chloride
D ) oral fludrocortisone
E ) mineralocorticoid replacement only when steroids weaned
F ) All of the above are true

The correct answer is choice C

Named for Thomas Addison, the physician who first described the condition in 1855, Addison
disease is adrenocortical insufficiency, due to dysfunction or destruction of the adrenal cortex.
Since this can occur gradually with symptoms developing only after about 90 percent of both
adrenal glands is destroyed, Addison disease tends to present between the ages of 30-50
years. Though it is a common topic on exams, it is a very rare disease, with an incidence today
of 40- 60 cases per 1 million people. It was more common in the past as a sequela of
tuberculosis (TB) which can destroy the adrenal glands, and several famous individuals have
had it, most notably US President John F Kennedy. Certain authors such as Charles Dickens
and Jane Austen have been speculated to have had Addison disease as well.

When not the result of TB, Addison disease is thought to develop due to atrophy, lymphocytic
infiltration, and fibrosis of the adrenal cortex, resulting from autoantibodies, generally with
no destruction of the adrenal medulla, and patients may be predisposed due to hereditary
factors. Due to a lack of cortisol, patients may exhibit nausea, vomiting, and fatigue as in the
case of this patient. Females may present with amenorrhea and men with impotence and
decreased libido. Patients are hypotensive and may demonstrate orthostatic intolerance and
complain of dizziness as well. If severe this can lead to presyncope and syncope.
Hypoglycemia is to be expected due to lack of cortisol. In diabetic patients, patients also may
become hypoglycemic due to increased sensitivity to insulin.

Reduced cortisol leads to an increase in pituitary secretion of ACTH, which stimulates


melanocytes in the skin to produce melanin. Generally, this effect precedes the other
symptoms, since for a time the increased ACTH can help to squeeze a little more cortisol out
of the deteriorating adrenal glands. Eventually, however, weakness, fatigue, reduced appetite,
and weight loss develop progressively. Hyperkalemia results from the lack of cortical
hormones and this leads to myalgias and flaccid muscle paralysis.

Since blood pressure, volume, and electrolyte balance needs to be restored, IV infusion of
isotonic sodium (thus choice C is incorrect) is necessary immediately, while IV hydrocortisone
(choice A) will provide the patient with an immediate burst of the deficient hormone that is
causing her condition. Subsequently, you will need to get her started on oral fludrocortisone
(choice D). A variety of other autoimmune conditions may occur in association with Addison
disease and these include the autoimmune thyroid conditions Hashimoto thyroiditis and
Graves disease. For this reason, it is important to carry out thyroid function tests (choice B).

As long as the patient is receiving 100 mg or more of hydrocortisone in 24 hours, no


mineralocorticoid (choice E) replacement is necessary. The mineralocorticoid activity of
hydrocortisone in this dosage is sufficient.

A 74-year-old man with low ejection fraction systolic heart failure comes to the Emergency
Department with shortness of breath. His past medical history is significant for small cell lung
cancer and he is on metformin. On physical exam, the patient is found to have 2+ pitting
edema in both lower extremities. The patient is started on furosemide. His metabolic panel is
within normal limits with the exception of serum sodium of 128 mEq/L.

Syndrome of inappropriate antidiuretic hormone production (SIADH) is characterized by


which of the following?

A ) hyponatremia/low serum osmolality/hyperkalemia


B ) hypernatremia/high serum osmolality/hypokalemia
C ) hyponatremia/normal urinary osmolality
D ) hypokalemia/high urinary osmolality
E ) hyperkalemia/low urinary osmolality

The correct answer is choice C

The patient!s euvolemic status and hyponatremia suggests that SIADH is the most cause of of
his hyponatremia. Syndrome of inappropriate antidiuretic hormoneis paraneoplastic
syndrome commonly associated with small cell carcinoma of the lungs but the hyponatremia
is euvolemic. Patient!s would not present with physical findings of volume overload (i.e.
ascites, pitting edema).

Hyponatremia is an electrolyte disturbance in which the sodium concentration in the serum is


lower than normal. Sodium is the dominant extracellular cation and cannot freely cross the
cell membrane. Normal serum sodium levels are between 135-145 mEq/L. Hyponatremia is
defined as a serum level of less than 135 mEq/L and is considered severe when the serum
level is below 125 mEq/L.

Hyponatremia is most often a complication of other medical illnesses in which either fluids
rich in sodium are lost (diarrhea or vomiting) or excess water accumulates in the body at a
higher rate than can be excreted (for example in congestive heart failure, syndrome of
inappropriate antidiuretic hormone, SIADH, or polydipsia). Regarding sodium loss as a cause
of hyponatremia, it is important to note that such losses promote hyponatremia in only an
indirect manner. In particular, hyponatremia occurring in association with sodium loss does
not reflect inadequate sodium availability as a result of the losses. Rather, the sodium loss
leads to a state of volume depletion, with volume depletion serving as signal for the release
of ADH (anti-diuretic hormone). As a result of ADH-stimulated water retention, blood sodium
becomes diluted and hyponatremia results.

Symptoms of hyponatremia include nausea and vomiting, headache, confusion, lethargy,


fatigue, appetite loss, restlessness and irritability, muscle weakness, spasms, or cramps,
seizures, and decreased consciousness or coma. The presence and severity of symptoms are
associated with the level of serum sodium, with the lowest levels of serum sodium associated
with the more prominent and serious symptoms.

Neurological symptoms often show for extremely low levels of sodium. When sodium levels in
blood become too low, excess water enters cells and causes the cells to swell. Swelling in the
brain is especially dangerous because the brain is confined by the skull and is unable to
expand. Neurological symptoms most often are due to very low serum sodium levels (usually
<115 mEq/L), resulting in intracerebral osmotic fluid shifts and brain edema. This neurological
symptom complex can lead to tentorial herniation with subsequent brain stem compression
and respiratory arrest, resulting in death in the most severe cases.

The imbalance between sodium and water in blood may occur in three primary ways and is
based on the patient!s volume status:

Hypervolemic hyponatremia, excess water dilutes the sodium concentration, causing low
sodium levels. Hypervolemic hyponatremia is commonly the result of kidney failure, heart
failure or liver failure.

Euvolemic hyponatremia, normal water levels are combined with low sodium levels. This
condition is commonly due to chronic health conditions, cancer or certain medications.

Hypovolemic hyponatremia, water and sodium levels are both low. This may occur, for
example, when exercising in the heat without replenishing fluid electrolytes or with marked
blood loss.

hyponatremia/low serum osmolality/hyperkalemia (choice A) is incorrect as SIADH has no


effect or serum potassium or volume status.

hypernatremia/high serum osmolality/hypokalemia (choice B) is incorrect as SIADH has no


effect or serum potassium or volume status.

hypokalemia/high urinary osmolali (choice D) is incorrect as SIADH has no effect or serum


potassium or volume status.
hyperkalemia/low urinary osmolality (choice E) is incorrect as SIADH has no effect or serum
potassium or volume status.

A 35-year-old man admitted to hospital due to chronic cough and low-grade fever was
diagnosed to have a pulmonary tuberculosis (PTB). On admission, laboratory tests showed
seropositivity for HIV. He begins therapy with isoniazid, rifampicin, pyrazinamide, and
ethambutol. The patient is observed to have a prompt clinical response to this multi-drug
treatment.

Two weeks after discharge, further evaluation revealed a CD4+ cell count of 52/mm3 and a
plasma HIV RNA level of 500,000 copies/mL. Antiretroviral therapy consisting of zidovudine,
lamivudine, and indinavir is initiated. Rifampin is switched to rifabutin. Three weeks later, he
develops high fever and is readmitted to hospital. His chest radiograph now reveals
intrathoracic lymphadenopathy and worsening of lung infiltrates. Sputum tests for Gram
staining, and acid-fast bacilli as well as blood culture are negative for any organisms.

Which of the following is the most likely diagnosis?

A ) Drug-resistant tuberculosis
B ) Immune reconstitution
C ) Non-Hodgkin’s lymphoma
D ) Rifabutin hypersensitivity
E ) Lamivudine hypersensitivity

The correct answer is choice B.

Regarding patients with co-infections of TB and HIV, active TB develops in 5-10 years in HIV-
infected patients with known latent tuerculosis infection. In comparison, persons who are not
immunocompromised have about the same percentage over a lifetime. Morbidity is high in
HIV-infected TB patients who have either untreated or infected with multiple drug resistant
strain. In the early 1990s, half of this population died with a median survival of only 60 days.
Outcomes are a little better now in developed countries because diagnosis is earlier and
because of the development of antiretroviral therapy, but still remains a serious concern. In
developing countries, mortality is high. Bacillary dissemination during primary infection is
more extensive in patients with HIV infection thus, a bigger proportion of TB is
extrapulmonary. Smear-negative TB is more common when HIV co-infection is present.
Immune reconstitution inflammatory syndrome (IRIS) is seen in some cases of HIV or any
condition causing immunosuppression, when the patient!s immune sytem begins to recover
from tuberculosis but then paradoxically responds to a previous acquired opportunistic
infection with overwhelming inflammatory response that worsens the symptoms of the
original infection. Suppression of CD4 T cells by the HIV infection decreases the normal
response to fight certain infections and the patient becomes symptomatic. On effective HIV
treatment, the CD4 count can rapidly increase, allowing for a more effective immune
response to inflammation thus the production of nonspecific symptoms such as high fever.
The patient in this case shows a paradoxical symptomatic relapse of the tuberculosis infection
despite successful microbiological treatment. Microbiologic cultures are sterile. The causes
are not well known but it is hypothesized that reconsitution of antigen-specific T
cell-mediated immunity with activation of immune system following HIV therapy against the
persisting antigen whether present as intact organisms, dead organisms or debris. Though
these symptoms can pose a danger to the patient, this could also indicate that the person!s
immune system has a better chance of fighting the infection. The events usually
spontaneously can get better with time without additional management but in severe cases
anti-inflammatory medications may be indicated to suppress inflammation until the infection
is considered eliminated.
The patient most likely would not have drug resistant tuberculosis (Choice A) in this case as
the patient had been observed to get clinically better. This improvement would not occur if
the tuberculosis strain is resistant to the multiple therapy.
Like primary HIV infection, Non Hodgkin!s Lymphoma (Choice C) usually presents with
painless lymphadenopathy or when mediastinal adenopathy is detected on routine chest
x-ray. Chest x-ray findings may be similar to tuberculosis, lung carcinoma, or sarcoidosis.
Like rifampin, rifabutin is efficacious in tuberculosis regimen in HIV positive patients. In this
patient!s situation, rifabutin hypersensitivity (Choice D) or lamivudine hypersensitivity (Choice
E) would probably manifest earlier than three weeks. Pyrexia, rash and other hypersensitivity
reactions such as eosinophilia can occur rarely with rifabutin administration as well as other
antibiotics. Lamivudine is a synthetic nucleoside analogue with activity against hepatitis B and
HIV. Reactions toward lamivudine were usually dizziness, depressive disorders, pancreatitis,
neutropenia and elevations in liver function tests. Reference: Shelburne S et al. Incidence and
risk factors for immune reconstitution inflammatory syndrome during highly active
antiretroviral therapy.

A 22 year old female presents to a university health clinic with a 3-day history of urinary
urgency and frequency. She states that she has had two prior urinary tract infections and
feels she has developed another one. Urinalysis confirms the diagnosis of cystitis and you
recommend a 3-day course of ciprofloxacin.

The mechanism of action of ciprofloxacin includes which of the following:

a) Inhibition of cell wall synthesis.


b) Inhibition of topoisomeras II/IV
c) Inhibition of ribosomal protein synthesis
d) Inhibition of mitochondria electron transfer
e) Inhibition of folic acid metabolism

The correct answer is choice B

The quinolone class of antibiotics performs its bacterial killing in a concentration-dependent


fashion via its actions on cellular DNA—specifically by inhibiting DNA gyrase. This results in
the inability of the cell to replicate, resulting in the bacterial population’s demise.

Inhibition of cell wall synthesis (Choice A) is the mechanism of action of beta-lactams and
glycopeptides.

Inhibition of ribosomal protein synthesis (Choice C) is the mechanism of action of the


tetracyclines, chloramphenicol, aminoglycosides and macrolides.

Inhibition of mitochondria electron transfer (Choice D) is the mechanism of action of


antimalarials.

Inhibition of folic acid metabolism (Choice E) is the mechanism of action of sulfonamides.

Review

Macrolides and Clindamycin ----> blocking the trans-location step of translation (50S subunit);
streptogramins may also work on the extrusion of the completed peptide.

Aminoglycosides ----> Prevent initiation complex by causing misreading of the mRNA code
(30S subunit)

Chloremphenicol and cycloheximides ----> inhibit peptidyl transferase (50S subunit)

Tetracyclines, Streptogramins, and Linezolid ----> inhibit initiation complex formation by


inhibiting the attachment of tRNA to the A site (30S subunit for tetracylin: 50S subunit for
streptogramins and linezolid)

Pencillins and cephalosporins ----> bind to PBP and inhibit transpeptidation of the cell wall

Flouroquinolones ----> inhibit topoisomeras II (DNA gyrase) and topoisomerase IV

Sulfonamides ----> blocking dihydropterate synthetase

Trimethoprim and pyrimethamine ----> blocks dihydrofolate reductase

Vancomycin ----> binds to D-Ala-D-Ala muramyl pentapeptide and halt transglycosylation of


the peptidoglycan elongation in bacterial cell wall

Metronidazole ----> free radicals formation (needs oxygen to work)

Polymyxins ----> damages the membrane osmotic properties

INH ----> inhibit synthesis of mycolic acid


Rifampin ----> works on sigma factor needed for initiation of transcription

Ethambutol ----> inhibit synthesis of arabinogalactan a cell wall component of mycobacterium

Puromycin ----> attaches to the A site leading to premature termination of translation

Some easy sayings to memorize:

"Buy AT (Aminoglycosides, Tetracyclines) 30, CEL (Clindamycin, Erythromycin, MacroLides) At


(Azithromycin) 50".

Which antibiotics are BACTERIOSTATIC:

Linda (cLindamycin) had a BMT (Bactrim, Macrolides, Tetracyclin) and is Stable (Static) (the
rest are mainly BACTERIOCIDAL).

A 42-year-old woman has noted increasing dyspnea for the past 6 years. On examination rales
are auscultated in both lungs. She is afebrile. A chest radiograph shows an enlarged cardiac
silhouette and bilateral pulmonary edema. Past history reveals that, as a child, she suffered
recurrent bouts of pharyngitis with group A beta hemolytic streptococcal.

Which of the following cardiac valves are most likely to be abnormal in this woman?

a) Aortic and tricuspid


b) Mitral and pulmonic
c) Aortic and pulmonic
d) Tricuspid and pulmonic
e) Mitral and aortic

The correct answer is choice E.

This is a case of rheumatic heart disease complicated with congestive heart failure (enlarged
cardiac shadow and pulmonary edema).

Rheumatic heart disease is the most serious complication of acute rheumatic fever
(evidenced in this case by the history of recurrent bouts of pharyngitis with group A beta
hemolytic streptococci.

In rheumatic heart disease the mitral valve is affected most commonly and severely (65-70%
of patients); the aortic valve is affected second most commonly (25%).The tricuspid valve is
deformed in only 10% of patients, almost always in association with mitral and aortic lesions,
and the pulmonary valve is rarely affected.

Rheumatic fever (RF) is a systemic illness that may occur following group A beta hemolytic
streptococcal (GABHS) pharyngitis in children. It is thought to result from an inflammatory
autoimmune response. The current incidence of rheumatic fever after GABHS infection is now
thought to have decreased to less than 1%.

The proposed pathophysiology for development of rheumatic heart disease is as follows:


Cross-reactive antibodies bind to cardiac tissue facilitating infiltration of streptococcal-primed
CD4+ T cells, which then trigger an autoimmune reaction releasing inflammatory cytokines
(including TNF-alpha and IFN-gamma).

A diagnosis of rheumatic heart disease is made after confirming antecedent rheumatic fever.
The modified Jones criteria (revised in 1992) provide guidelines for the diagnosis of rheumatic
fever. The Jones criteria require the presence of 2 major or 1 major and 2 minor criteria for
the diagnosis of rheumatic fever:

The major diagnostic criteria include carditis, polyarthritis, chorea, subcutaneous nodules,
and erythema marginatum.

The minor diagnostic criteria include fever, arthralgia, prolonged PR interval on ECG, elevated
acute phase reactants (increased erythrocyte sedimentation rate [ESR]), presence of
C-reactive protein, leukocytosis, and history of beta hemolytic streptococcal pharyngitis.

A 7 year old male is brought to the emergency department by his grandmother for evaluation
of altered mental status. You assess the patient, noting a somnolent child who moans in
response to noxious stimuli. Vital signs include temperature 37.1 C, pulse 132 bpm,
respirations 20/min, blood pressure 174/100, and oxygen saturation 94% in room air. As you
begin the physical examination, the patient experiences a generalized, tonic-clonic seizure.
Seizure activity stops following a dose of intravenous lorazepam.

Which of the following is the most appropriate next step in the management of this patient?

a) Computed tomography (CT) of the head without intravenous contrast


b) Lumbar puncture
c) Sublingual nifedipine
d) Intravenous fosphenytoin
e) Intravenous nitroprusside
The correct answer is choice E.

This patient has a hypertensive emergency, defined as severe hypertension associated with
evidence of end-organ damage. Treatment with an intravenous antihypertensive agent like
nitroprusside (choice E), hydralazine, labetalol, or nicardipine should be initiated immediately,
with the goal of lowering the blood pressure by no more than 25% over the first several hours.

A CT scan of the head without i.v. contrast (choice A) is a rapid and useful test for identifying
acute intracranial bleeding. It would be useful to obtain this test as quickly as possible in a
patient like the one described - but not before he is stabilized! Sending a patient with
unstable vital signs to radiology would be a very dangerous decision - you should treat this
patient's severe hypertension first.

A lumbar puncture (choice B ) could be useful in evaluating new-onset seizures, as it would


identify infections like meningitis or encephalitis. However, this patient has a more likely
reason to seize - his blood pressure. Moreover, performing an LP without first ruling out
increased intracranial pressure puts the patient at risk for cerebral herniation and death.

Sublingual nifedipine (choice C) was commonly used in the treatment of hypertensive


urgencies and emergencies before evidence emerged that, at least in adults, this treatment
could be harmful. In 1996, the U.S. FDA recommended not using nifedipine for this purpose.
The problems seem to occur because the initial drop in blood pressure can be too great,
causing decreased perfusion to the heart or brain. Treatment with rapidly acting intravenous
agents is now preferred, as these can be more easily titrated minute-by-minute to prevent
excessive drops in BP.

Fosphenytoin (choice D) would be the agent of choice for status epilepticus that does not
respond to first-line therapy with benzodiazepines. However, this patient's seizures (for the
moment, at least) have stopped. Giving additional anticonvulsants will not be as helpful as
treating the cause of this patient's seizure.

A 65- year- old retired truck driver who smokes a pack of cigarettes daily is diagnosed with
essential hypertension. He has no previous history of cardiovascular disease or sexual
dysfunction. In the primary care physician’s office, his blood pressure is 170/95 mmHg. He is
prescribed a single therapeutic agent. Shortly after the beginning of the treatment, he
experiences difficulties in obtaining and maintaining an erection.

What medication is most likely to have caused this adverse effect?

a) Hydrochlorothiazide
b) Enalapril
c) Losartan
d) Prazosin
e) Verapamil

The correct answer is choice A.

Many classes of antihypertensive medications have been associated with erectile dysfunction
(ED). Hydrochlorothiazide is a thiazide diuretic. Among the antihypertensive medications,
thiazides and beta-blockers are the most common cause of ED. Among thiazides,
chlorthalidone is considered to have the strongest effect on sexual function.
Hydrochlorothiazide has a moderate ED- inducing effect, which however might be
dose-dependent.

Thiazide diuretics inhibit Na+/Cl− re-absorption from the distal convoluted tubules by
blocking the thiazide- sensitive Na+/Cl− symporter. They are recommended as first line
treatment for hypertension in the US (The Seventh Report of the Joint National Committee on
Prevention, Detection, Evaluation, and Treatment of High Blood Pressure, JNC VII, guidelines),
as well as in the National Institute of Health and Clinical Excellence/British Hypertension
Society guidelines and in the European (ESC/ESH) guidelines. Their adverse effects on male
sexual function have been reported in several studies, with an incidence that varies between
3 and 32%. These effects are probably due to an action on vascular smooth muscle. In a study
of 15,000 hypertensive patients receiving bendrofluazide or the beta-blocker propranolol, for
5 years, it was found that impotence was 22-fold and 4-fold higher (compared to placebo) in
patients treated with thiazide and beta-blocker respectively. Impotence was also the principal
reason for withdrawal from antihypertensive therapy.

Angiotensin- converting enzyme (ACE) inhibitors such as Enalapril (choice B ) have a very low
likelihood of causing ED.

Losartan (choice C ), an angiotensin II receptor antagonist, is unlikely to cause ED. On the


contrary, it may be beneficial in this condition.

As an alpha-blocker, Prazosin (choice D), has a low likelihood of inducing ED.


Calcium channel blockers such as Verapamil (choice E) are known to have a low incidence of
sexual side effects.

Review:

Many drugs have been reported to cause erectile dysfunction. Central neurotransmitter
pathways, including serotonergic, noradrenergic, and dopaminergic pathways involved in
sexual function, may be disturbed by antipsychotics, antidepressants and centrally acting
antihypertensive drugs. Beta-adrenergic blocking drugs may cause erectile dysfunction by
potentiating alpha-1 adrenergic activity in the penis. Thiazide diuretics have been reported to
cause erectile dysfunction, but the cause is unknown. Spironolactone can cause erectile
failure as well as decrease in libido and gynecomastia. Cigarette smoking may induce
vasoconstriction and penile venous leakage because of its contractile effect on the cavernous
smooth muscle. Alcohol in small amounts improves erection and increases libido because of
its vasodilatory effect and the suppression of anxiety; however, large amounts can cause
central sedation, decreased libido and transient erectile dysfunction. Cimetidine, a
histamine-H2 receptor antagonist, has been reported to decrease libido and cause erectile
failure; it acts as an antiandrogen. Other drugs known to cause erectile dysfunction are
estrogens and drugs with antiandrogenic action such as ketoconazole and cyproterone
acetate.

A one month old baby girl is seen by a pediatrician for an unexplainable rhinorrhea and poor
feeding. The girl was born after seven months of gestation. At birth no significant
abnormalities have been noted except for a prominent jaundice and a mild hepatomegaly.
The mother is of low socio-economical status who has poorly followed pre and post-natal
appointments. A congenital syphilis is suspected.

All of the following are primary lesions associated with congenital syphilis, EXCEPT:

a) nephrosis
b) cataract
c) hydrops
d) ascites
e) mulberry molars

The correct answer is choice B


Cataract has been seen in syphilis cases; however, whether the development is caused by T
pallidum (primary) or from the uveitis associated with the disease (secondary cataract) is
unknown. Primary congenital cataracts of infectious origin are usually caused by Rubella,
Varicella and CMV viruses.

The following are the most common specific presentations of congenital syphilis:

HEAD
Hutchinson's triad (deafness + Hutchinson's teeth + interstitial keratitis) Mulberry molars
(sixth year molars with multiple poorly developed cusps). Frontal bossing

Poorly developed maxillae ("bulldog" face) Snuffles (rhinitis with mucopurulent or


blood-stained discharge) saddle nose deformity Petechiae and other skin rashes (macular,
copper-colored rash on the palms and soles) Rhagades (linear scars around the mouth/ nose/
anus) Jaundice Fetal hydrops (skin edema + fluid in two other compartments: ascites, pleuritis,
pericarditis)

EXTREMITIES
Sabre shins Pseudoparalysis (as a result of osteochondritis) Higoumenakis sign (enlargement
of the sternal end of clavicle in late congenital syphilis)

INTERNAL ORGANS
Enlarged liver
Enlarged spleen

HEMATOLOGICAL
Anemia
Thrombocytopenia
Lymph node enlargement

An 80 year-old male presents with lethargy and anorexia for three days duration. He has a
history of atrial fibrillation and has been rate controlled for several years with digoxin. His
history is otherwise unremarkable, and his physical exam is within normal limits. After labs
are drawn he is found to have a plasma potassium level of 6.0 mEq/L and a digoxin level of 4.0
(normal therapeutic levels are between 0.8 - 2.0).

Each of the following may be a direct consequence of severe magnesium deficiency except:
a) digitalis induced arrhythmias
b) hypocalcemia
c) hypokalemia
d) hyponatremia
e) confusion

The correct answer is D


Magnesium is needed for the adequate function of the Na+/K+-ATPase pumps in the cells of
the heart. A lack of it depolarises and results in tachyarrhythmia. Magnesium intravenously
helps in refractory arrhythmia, most notably torsade de pointes. Others are ventricular
tachycardia, supraventricular tachycardia and atrial fibrillation. The effect is based upon
decreased excitability by depolarisation and the slowing down of electric signals in the
AV-node. Magnesium is a negative inotrope as a result of decrease calcium influx and calcium
release from intracellular storage. It is just as effective as verapamil. In myocardial infarction
there is a functional lack of magnesium, supplementation will decrease mortality.

Deficiency of magnesium causes weakness, muscle cramps, cardiac arrhythmia, increased


irritability of the nervous system with tremors, athetosis, jerking, nystagmus and an extensor
plantar reflex. In addition, there may be confusion, disorientation, hallucinations, depression,
epileptic fits, hypertension, tachycardia and tetany.

Magnesium deficiency is not uncommon in hospitalized patients. Elevated levels of


magnesium. 10-20% of all hospital patients and 60-65% of patient in the intensive care unit
have hypomagnesemia. Hypomagnesemia is underdiagnosed, as testing for serum
magnesium levels is not routine.

Hypomagnesemia results in increased mortality. Treatment of hypomagnesemia depends on


the degree of deficiency and the clinical effects. Oral replacement is appropriate for patients
with mild symptoms, while intravenous replacement is indicated for patients with severe
clinical effects.

Magnesium deficiency has no effect on serum sodium levels.


Digitalis-induced Arrhythmias (choice A) can result from magnesium deficiency. Magnesium is
needed for the adequate function of the Na+/K+-ATPase pumps in the cells of the heart. A
lack of it depolarises and results in tachyarrhythmia. Magnesium inhibits release of potassium,
a lack of magnesium increases loss of potassium. Intracellular levels of potassium decrease
and the cells depolarise. Digoxin increases this effect. Both digoxin and hypomagnesemia
inhibit the Na-K-pump resulting in decreased intracellular potassium.

Hypocalcemia (choice can result from magnesium deficiency. Release of calcium from the
sarcoplasmic reticulum is inhibited by magnesium. Low levels of magnesium stimulate the
release of calcium and thereby an intracellular level of calcium. This effect similar to calcium
inhibitors makes it "nature's calcium inhibitor." Lack of magnesium inhibits the release of
parathyroid hormone, which can result in hypoparathyroidism and hypocalcemia.
Furthermore, it makes skeletal and muscle receptors less sensitive to parathyroid hormone.

Hypokalemia (chocie C) can result from magnesium deficiency. Potassium channels are
inhibited by magnesium. Hypomagnesemia results in increased efflux of intracellular K. The
cell loses potassium which then is excreted by the kidneys, resulting in hypokalemia.

Confusion (choice E) can result from magnesium deficiency. The neurological effects of
magnesium deficiency are due to reduced electrical excitation and blocked release of
acetylcholine and N-methyl-D-aspartate (NMDA) glutamate receptors, an excitatory
neurotransmitter of the central nervous system.

An HIV positive male with a history of IV drug abuse and schizophrenia is brought to the
emergency department in a confused, incoherent and agitated state. Vitals are pulse 118, BP
160/90, respiratory rate 15, and temperature is 37.5 celcius orally. His skin is hot and flushed
and you noticed he has dilated pupils. The physical examination is otherwise unremarkable.
Medical records indicate that his regular medications include chlorpromazine, cogentin, and
acyclovir.

The most likely diagnosis is which of the following conditions:

a) Opiod toxicity
b) Anticholinergic intoxication
c) Barbiturates toxidrome
d) Cocaine toxicity
e) Cholinergic toxidrome

The correct answer is B.

This patient exhibits evidence of the anticholinergic toxidrome. This syndrome is best
described as "hot as a hare, dry as a bone, and mad as a hatter." It also produced pupil
dilation, urinary retention (an important differentiating factor compared with
sympathomimetic syndromes), and seizures.

He is currently taking two medications (chlorpromazine and cogentin) that produce


anticholinergic symptoms.

You are working in a local TB health center and you are about to interpret the result of a
Mantoux tuberculin skin test on one of your patients who has had a history of fever and
cough.

False negative tuberculin test may be seen in all of the following, except which one?

a) Miliary TB
b) Immunosuppressive therapy
c) TB meningitis
d) Elderly patients
e) HIV infection

The correct answer is choice C.

The tuberculin test can identify patients with latent TB. False negative tuberculin tests occur
when patients do not react to the injected tuberculin purified protein derivative (PPD), even
though they are infected with M. tuberculosis. According to the CDC, false-negative reactions
can occur in the following settings (this list is not exhaustive):

Cutaneous anergy - inability to react to skin tests due to immunosuppression

Recent TB infection (within 8-10 weeks of exposure) - the test relies on the development of
cell-mediated immunity (CMI)

Very old TB infection (many years)

Patients less than 6 months old and the elderly, through decreased CMI
Recent immunization with a live-virus vaccine (e.g. smallpox, measles, mumps, polio)

Overwhelming TB disease e.g. miliary

Viral infections (e.g. measles and chicken pox)

Metabolic disease including malnutrition and CKD (again, impaired CMI)

Incorrect test administration

Incorrect reading of the test

These tests must be performed and interpreted in a standardized way.

Inject 0.1 ml of tuberculin purified protein derivative (PPD) into the inner surface of the
forearm

Use a tuberculin syringe and the bevel of the needle should be facing upwards

Make an intradermal injection - this should produce a pale wheal 6 to 10 mm in diameter

The skin test reaction should be read 48-72 hours after injection

You will be measuring the size of the area of induration (palpable, raised, hardened area or
swelling) NOT erythema, and it is the diameter in mm ACROSS the forearm, not along it.

The interpretation of the test result depends not just on the size of the induration but also on
the risk of infection and progression of disease

An 18 year-old college freshman notices difficulty in removing and replacing his shoes at the
end of the day and his face appears to be "puffy" and swollen. Over the ensuing two weeks he
develops noticeable swelling of his legs all the way up to his thighs. Other than tonsillectomy
at age 3, he has never had any medical problems and takes no medications.

On physical examination at the health center, his blood pressure is 118/72, pulse 76,
respiration 12, and he is afebrile. Weight is 80 kilograms, an increase of 7 kilograms over his
usual weight . He appears well, with no evidence of acute illness. The only finding on physical
exam is a 3+ to 4+ pitting edema extending all the way up his legs into the presacral area.

Laboratory evaluation reveals:


CBC normal

sodium 140 mmol/L, potassium 3.7 mmol/L, chloride 101 mmol/L, CO2 29 mEq/L

BUN 17 mg/dL (6 mmol/L) [normal 8-25 mg/dL or 2.5-9 mmol/L]

creatinine 1 g/dL (88.4 μmol/L) [normal 0.6-1.5 mg/dL or 53-132.6 μmol/L]

urinalysis: pH 5, specific gravity 1.012, 4+ protein with no blood or glucose

urine microscopy: occasional oval fat bodies and rare hyaline casts

Additional labs:

serum albumin 1.1 g/dL (11 g/L) [normal 3-5 g/dL or 35-50 g/L]

calcium 7.9 mg/dL (2 mmol/L) [normal 8.5-10.3 mg/dL or 2.1-2.6 mmol/L]

phosphorus 3.3 mg/dL (1.07 mmol/L) [normal 3-4.5 mg/dL or 0.97-1.45 mmol/L]

total cholesterol 393 (10.16 mmol/L) [normal < 200 mg/dL or < 5.2 mmol/L]

24-hour urine: protein 13.2 g, creatinine 2.248 g

Complement levels: normal

Serology: ANA, HBV, HCV negative

What IS YOUR PRELIMINARY diagnosis?

a) Nephritic syndrome
b) Nephrotic syndrome
c) Acute kidney injury
d) A and C
e) B and C

The correct answer is choice B.

This young man has nephrotic syndrome, which is characterised by:

heavy proteinuria (> 3.5 g / 1.73m2 / day)

hypoalbuminemia due to urinary protein losses

edema due to salt and water retention and reduced oncotic pressure

hyperlipidemia

lipiduria

There are a number of pointers to his not having nephritic syndrome:


his urinary protein losses are too high

he does not have acute kidney injury

he does not have hematuria

he has severe hyperlipidemia which is not a component of nephritic syndrome

Nephrotic syndrome results from glomerular damage which leads to an increase in


glomerular permeability. This causes proteins and other large substances that would normally
not be filtered to pass into the urine. Although albumin is easily measured and most talked
about, you should not forget that other proteins including immunoglobulin and complement
proteins are often being lost as well.

Nephrotic syndrome results from primary or secondary glomerular damage, primary causes
being diagnoses of exclusion when no secondary casue can be found. Pathologically the
glomerular manifestations include:

focal segmental glomerulosclerosis (FSGS) in up to 35% of cases

membranous nephropathy in up to 33% of cases

minimal change disease in up to 15% of cases

membranoproliferative glomerulonephritis in up to 14% of cases

Secondary causes include:

Diabetes mellitus

Infections such as hepatitis B & C, HIV, mycoplasma

Sjogren's syndrome

SLE

Sarcoid

Malignancy including lymphoma and myeloma

Obesity

Drugs such as gold/penicilliamine/NSAIDs/antibiotics/tamoxifen

Patients should be referred to a nephrologist for investigation and likely renal biopsy. While
this is pending, management often involves salt and fluid restriction, diuretics for edema,
statins for hyperlipidemia, and angiotensin ceonverting enzyme inhibitors in an attempt to
reduce protein spillage.
Nephrotic syndrome patients are at risk of DVT (1.5%) or renal vein thrombosis (0.5%), as well
as infection (loss of immunoglobulins and complement).

A 65 year-old male presents with a 4-6 month history of intermittent production of red-tinged
urine. Physical exam reveals a well nourished patient with slightly enlarged prostate and no
other abnormal findings.

Initial laboratory data reveals:

hemoglobin 18.7 g/dl (normal: 13.0-18.0 g/dL)


hematocrit 58% (normal: 37-49%)

BUN 10 mg/dl = urea of 3.6 mmol/L (normal: 8-25 mg/dL or 2.9-8.9 mmol/L)

creatinine 0.8 mg/dl = 70 μmol/L (normal: 0.6-1.5 mg/dL or 52-132 μmol/L)

calcium 12.3 mg/dl = 3.1


mmol/L (normal: 8.5-10.5 mg/dL or 2.1-2.6 mmol/L)
urinalysis: ++ hematuria, PMN.

The patient's lab results are most consistent with which of the following?

a) Dehydration
b) Paraneoplastic syndrome
c) Renal hematoma
d) Cystitis secondary to E. coli
e) Renal calculi

The correct answer is choice B.

This patient has a combination of polycythemia (erythrocytosis), hematuria, and


hypercalcemia. His urea/BUN is in the normal range and this excludes dehydration as the
cause of his elevated hematocrit, and it would not explain his intermittent hematuria.

Renal hematoma generally follows trauma (non-traumatic renal hematoma can occur but it
rare), and there is no suggestive history here. Like cystitis and renal calculi this diagnosis does
not explain the hypercalcemia or the erythrocytosis.

The combination of laboratory findings is compatible with a diagnosis of paraneoplastic


syndrome. Although hypercalcemia can occur in many tumor syndromes, the presence of
hematuria suggests that this patient may have a renal tumor, and renal cell carcinoma (RCC)
would be high up the list of differentials.

RCC originate from renal tubular epithelium and account for 95% of all renal tumors and
around 2-4% of all malignancies. Around 30% of RCC patients present with metastatic disease,
most often lung followed by soft tissue, bone, liver, skin and central nervous system. 30% of
RCC patients have a paraneoplastic syndrome.

There are many types of paraneoplastic syndrome, often categorized on the basis of the
predominant organ system affected. Mechanistically, they result from either:

secretion of biologically active substances

alterations of immunity e.g. auto-immune syndromes, immune complex disease

In some cases no mechanism can be determined. In the case of our patient, the
erythrocytosis is likely the result of ectopic erythropoietin secretion by the tumor cells. This
can be found in up to 10% of cases. There are two main mechanisms for the hypercalcemia;
humoral hypercalcemia of malignancy (HHM) due to secretion of
parathyroid-hormone-related-peptide (PTHrP) and osteolytic hypercalcemia. HHM is the most
common paraneoplastic syndrome in patients with RCC, occurring in 20% of cases.

A 32 year old man comes to the physician because of a long-standing belief that he is being
followed by foreign intelligence agents. He has not had hallucinations of any type. On mental
status examination, no formal thought disorder or change in mood is detected. His level of
functioning at work has remained high.

Which of the following is the most likely diagnosis?

a) obsessive compulsive disorder


b) panic disorder
c) schizophrenia paranoid type
d) schizotypal personality disorder
e) delusional disorder, persecutory type

The correct answer is E

Delusions are fixed false beliefs from which a person cannot be dissuaded. Persons suffering
from delusional disorder, persecutory type demonstrate at least 1 non-bizarre delusion but
can otherwise appear and be mentally normal and maintain high social and vocational
functioning. In contrast, persons with paranoid schizophrenia often possess bizarre delusions
and tend to develop social and vocational problems as a result of their behavior.
Obsessive compulsive disorder (Choice A) is incorrect because persons with obsessive
compulsive disorder (OCD) are not delusional. While their obsessions and compulsions may
seem odd, persons suffering from OCD have insight into their condition.

Panic disorder (Choice B) is incorrect because persons with panic disorder do not experience
delusions. Persons with panic disorder experience a discrete period of intense fear or
discomfort that develops abruptly, usually reaches a peak within 10 minutes, occurs in the
absence of real danger, and is accompanied by at least 4 of the following symptoms:
palpitations, pounding heart, or accelerated heart rate; sweating; trembling or shaking;
sensation of shortness of breath or smothering; feeling of choking; chest pain or discomfort;
nausea or abdominal distress; feeling dizzy, unsteady, lightheaded, or faint derealization
(feelings of unreality) or depersonalization (being detached from oneself); fear of losing
control or going crazy; fear of dying; paresthesias (unexplained numbness or tingling
sensations) and chills or hot flushes.

Schizophrenia, paranoid type (Choice C) is incorrect as per the discussion related to Choice A.
Additional symptoms that may be experienced by paranoid schizophrenia sufferers include
auditory or visual hallucinations, anxiety, anger, aloofness, verbal confrontations and
increased risk for suicidal thoughts and behavior.

Schizotypal personality disorder (Choice D) is not correct because persons with this
psychiatric disorder--while they may experience very strange thinking-- tend not to
experience delusions; their greatest concern is usually difficulty with interpersonal
relationships and disturbances in thought patterns, appearance, and behavior. The severity of
the behavior disturbance ranges from mild to severe, but those with marked disturbances
may resemble schizophrenics, making maintaining vocational and social relationships
problematic.

A male infant is admitted to your service on account of mental-motor deterioration with


convulsions and failure to thrive. The parents both are French Canadian, have an older child
who is healthy, and neither recalls any similar conditions occuring among family members. On
physical examination you find the head circumference to be in the 99th percentile for the
child's age. Though he is 8 months of age, he is unable to hold his head up. This is part of a
hypotonia which you find is generalized. On funduscopic examination, both of the child's
retinas appear similar to that in the image below.

Laboratory studies are likely to show:

a) mutation in band 15q 21.1 of chromosome 15


b) a 47,XY +18 karyotype

c) a 47,XY +21 karyotype

d) absence of serum hexosaminidase A activity

e) mutation on band 12q22-q24.1 of chromosome 12

The correct answer is choice D.


The failure to thrive, neuromotor deterioration, and the funduscopic finding (known as a
"cherry-red" spot on the macula)
presented in a child of this age are highly suggestive of Tay-Sachs disease. In this condition,
the lack of activity presented in a child of this age are highly suggestive of Tay-Sachs disease.
In this condition, the lack of activity of hexosaminidase A enzyme results in the accumulation
of GM2 ganglioside in lysosomes of neurons. This, in turn is caused by a mutation in the gene
HEXA. For the disease to manifest, one needs to have a mutation in both copies of HEXA,
since one functional gene is sufficient for metabolism of GM2; thus, Tay-Sachs is a recessive
disease. While HEXA a is located on chromosome 15, the correct band is 15q23-q24, located
in the long arm of the chromosome.

Band 15q 21.1 of the same chromosome (choice A) is the location of FBN1, the gene involved
in Marfan syndrome. Patients with this condition typically are tall with long extremities and
medical historians have speculated that figures such as Abraham Lincoln and the Egyptian
pharaoh Akhenaton may have had Marfan syndrome.
The karyotypes 47,XY +18 (choice B) and 47,XY +21 (choice C) are for Edward syndrome and
Down syndrome, respectively. While these trisomies both are associated with serious physical
as well as mental abnormalities, a very high proportion of concepti with these conditions are
eliminated by spontaneous abortion during early pregnancy. Infants born with Trisomy 18
have a very slim chance of surviving beyond one year and of those who survive the first few
months, their condition has been known since birth, or earlier. Of those with Down syndrome
who make it to birth, many actually are living into adulthood, since the notorious associated
physical abnormalities such as congenital heart defects and leukemia can be treated.
In contrast, absence of key metabolic enzymes such as occurs in diseases of the central
nervous system can take several months to manifest symptoms, as damage often results from
buildup of unmetabolized substances. Such is the case with Tay-Sachs as well as
phenylketonuria (PKU). A deficient ability to metabolize phenylalanine to tyrosine, results
from a mutation in the gene for the enzyme phenylalanine hydroxylase (PAH) which is located
on band 12q22- q24.1 of chromosome 12 (choice E).
In the case of PKU, however, the condition is treatable if detected early and patients can live
into adulthood with a diet restricted of phenylalanine. In the case of Tay-Sachs in its classic
infantile form, the condition is fatal, usually by ages 2- 4, since not treatment is available to
prevent the accumulation of GM2 ganglioside. Generally, macrocephaly (enlarged head,
defined as being above 97th percentile on head circumference) is noticeable by about 18
months of age, though it may occur earlier, as in the case of this child. In the course of
deteriorating, seizures, difficulty swollowing, spasticity, dementia, and blindness (choice A)
set in. Death usually results from pneumonia.
While there is an adult form of Tay-Sachs in which people have lived as long as 80 years, this
is a very different condition, in which HEXA activity, while low, is present nevertheless. This
results in the disease being chronic with symptoms appearing only during adulthood.
Additionally, there is a late infantile/juvenile form which is subacute, with onset between
ages 2-10 years.
As for the epidemiology of Tay-Sachs, typically what people retain from coursework is the
notorious incidence of the disease and the Tay-Sachs gene in Ashkenazic Jews. In North
America, it used to be that 99 percent Tay-Sachs diagnoses were in Jewish children, but the
success of genetic screening programs has reduced the incidence. At the same time, other
ethnic groups carry HEXA gene mutations of different types so that, internationally, 1 of every
280 people is a carrier for a HEXA mutation. Since the non-Jewish mutations were recognized
more recently than the one that affects Ashkenazic Jews, the proportion of Tay-Sachs
diagnoses in non-Jews has increased. In North America, non-Jewish populations with a high
incidence of Tay-Sachs include Pennsylvania Dutch (Amish), Louisiana Cajuns, and French
Canadians. Thus, in the eastern St. Lawrence River Valley of Quebec, incidence of Tay-Sachs
disease is similar to that of Jewish populations.

An agitated and combative 23 year old female is brought to the emergency department by
her concerned roommate. Laboratory evaluation shows:

Sodium 143 mEq/L

Potassium 4.3 mEq/L

Chloride 101 mEq/L

Bicarbonate 9 mEq/L

Blood urea nitrogen 18 mg/dL

Creatinine 0.8 mg/dL

Glucose 74 mg/dL

An arterial blood gas shows pH 7.33, pCO2 26 mm Hg, pAO2 103 mm Hg, and a base excess of
-15.

Which of the following is the most appropriate interpretation of this patient's acid-base
disorder?

a) Acute respiratory acidosis without evidence of compensation

b) Acute metabolic acidosis without evidence of compensation

c) Acute respiratory acidosis with simultaneous acute metabolic acidosis


d) Acute metabolic acidosis with partial respiratory compensation

e) Acute respiratory acidosis with partial metabolic compensation

The correct answer is choice D.

Even if you think you don't understand acid-base disorders or acid-base physiology, most
questions can be easily answered by recalling the bicarbonate buffer equation:

CO2 + H2O< --> H2CO3< --> H+ + HCO3

Bicarbonate is the body's major buffer system. A low serum bicarbonate means that this
patient has a metabolic acidosis, regardless of what other abnormalities may be present. Look
at the equation above: if you remove bicarbonate, the equation shifts to the right, generating
more protons and thus more acidosis.

The low pCO2 indicates that the patient has increased his or her minute ventilation in an
effort to "blow off" CO2 and create a respiratory alkalosis. This would be an appropriate
compensatory mechanism in the face of a primary metabolic acidosis. Again, look at the
equation above: if you remove CO2, the equation shifts to the left, removing protons (H+) and
generating alkalosis.

This patient, however, has compensated only partially. A good rule of thumb is that, in a
patient with metabolic acidosis, the pCO2 should drop by an amount that equals 1.2 times
the drop in the serum bicarbonate. Here, the serum bicarbonate dropped by about 15 (from a
normal bicarbonate of about 24 to 9). Multiply this by 1.2 and you get 18. Thus, if the patient
had been able to lower her pCO2 to about 22 or so (40 - 18), then the pH of the blood would
have returned to normal.

In a patient with an acute respiratory acidosis without evidence of compensation (choice A),
the pCO2 would be high, and the serum bicarbonate normal.

In a patient with an acute metabolic acidosis without evidence of compensation (choice b),
the serum bicarbonate would be low, and the pCO2 normal.

In a combined respiratory and metabolic acidosis (choice C), the pCO2 would be elevated,
while the bicarbonate would be low.

Acute respiratory acidosis (choice E) with partial metabolic compensation would result in an
elevated pCO2 with an elevated serum bicarbonate.
A 68 year old man is brought to the ER by EMS after being found unresponsive by his wife. He
underwent a radical resection of his bladder for bladder carcinoma a month ago and has been
experiencing persistent diarrhea since his discharge two weeks prior. His only other medical
issues are glaucoma (treated with acetazolamide) and previous myocardial infarction but a
complete history was not available.

On physical exam, the patient is unarousable even with sternal rubs and the rest of the exam
is unremarkable. His vitals are temperature of 37.2 degrees, BP 115/70, HR 95 and RR of 22. A
stat metabolic panel reveals:

Na: 137

K: 3.6

Cl: 95

HCO3: 20

BUN: 60

Cr: 3.4

Glucose: 342

Which of the following would explain the patient’s metabolic acidosis?

a) treatment of glaucoma

b) radical surgery of urinary bladder cancer

c) Persistent diarrhea

d) renal tubular acidosis type IV

e) diabetic ketoacidosis

A six year old girl comes to your office for her annual check up. On physical examination, you
note a normal S1 and S2 as well as a soft systolic murmur with a vibratory character, heard
best over the right upper sternal border. You suspect an innocent murmur, but her mother
asks if she should be referred to a cardiologist.

Each of the following findings would support the diagnosis of a pathological murmur EXCEPT:

a) the murmur occurs in diastole


b) the murmur is associated with a thrill
c) the murmur is louder in the supine position and softer when the child sits upright
d) the child has mild finger clubbing
e) there is a loud P2 component of the second heart sound

The correct answer is choice C.

Heart murmurs are common in children - as many as 50% of healthy children will have a
murmur detected at some point in childhood. Most murmurs in childhood are "innocent" and
not associated with significant cardiac disease. The child in the vignette has one such murmur:
a Still's murmur, classically described as having a "vibratory" or "musical" component.

Fundamentally, the presence of a heart murmur only indicates the existence of turbulent
blood flow. Most innocent murmurs are "flow murmurs", caused by increased blood flow
through normal cardiac anatomy. Children with fever or anemia have increased cardiac
output/blood flow and thus will have louder flow murmurs. Physical exam maneuvers, such as
listening to the heart in the supine and then sitting positions (choice C), will also increase the
intensity of a murmur. When the child is lying down, blood return to the heart is increased,
increasing cardiac preload. This results in increased cardiac output and a louder murmur. The
opposite situation occurs in adolescents with hypertrophic cardiomyopathy. In this illness, the
left ventricular outflow tract is partially obstructed by hypertrophied myocardium. When
stroke volumes are large, the tract is stented open by the blood flow - but when stroke
volume decreases, the obstruction worsens, leading to increased turbulence. When these
patients move from lying down to the sitting or standing position, their murmurs will be much
louder.

All innocent murmurs and flow murmurs are systolic. Any diastolic murmur (choice A) is
pathological and should be evaluated further.

Louder and harsher murmurs are also more likely to be pathologic. Any murmur associated
with a thrill (grade IV/VI or higher) is likely to be associated with cardiac disease (choice B ).

Symptoms of cardiac disease - such as digital clubbing (choice D) - that occur in the presence
of a murmur always merit further evaluation. Other symptoms of heart disease in infants and
children can include failure to thrive, cyanosis, or sweating or easily tiring out during feedings.

A loud P2 (choice E) indicates pulmonary hypertension, which results from many congenital
heart diseases that are associated with increased pulmonary blood flow.
The development of conduction disturbances due to myocardial infarction and arrhythmia
depends on the vascular supply of the different components of the heart conduction system.

All statements are correct about the blood supply of the conduction system of the heart
EXCEPT:

a) Sinoatrial node is is supplied by the right coronary artery in 60 percent of patients and by
the left circumflex artery in 40 percent of patients

b) Atrioventicular node supplied mostly by the right coronary artery

c) Main or proximal left bundle branch is mostly supplied by the left anterior descending
artery

d) Left posterior and anterior fascicles are supplied by the left anterior descending artery

e) Right bundle branch is supplied by the left anterior descending artery

You are looking after a 50 year old man who has severe acute pancreatitis secondary to biliary
tract stones. He develops chills and rigors, and spikes a fever of 39.5 Celsius. He is
complaining of increasing abdominal pain.

On exam he is jaundiced, and his blood pressure is 100/70 mmHg when supine falling to
90/60 mmHg when sitting, with an increase in heart rate from 95 to 120 beats per minute. His
jugular venous pressure appears to be low. His abdomen is distended and clinically consistent
with ascites. Over the next few hours his urine output falls from 50 ml/h to 15 ml/h.

Which of the following statements is FALSE?

a) This man is at high risk of acute kidney injury due to obstructive jaundice and hypovolemia

b) Urine sodium of 10 mmol/L would be consistent with ischemic ATN

c) He should receive boluses of isotonic saline or colloid until the JVP and BP are normalized,
with frequent review of the physical findings

d) At this time administration of furosemide to increase urine flow is contraindicated


e) Renal function should be monitored at least daily using serum creatinine

The correct answer is choice B.

The classical teaching is that a urinary sodium of < 20 mmol/L is supportive of a diagnosis of
hypovolemia or pre-renal oliguria, whereas ATN is classically associated with a urinary sodium
of > 40 mmol/L. The logic behind this is that in hypovolemia, the physiological response in
defense of intravascular volume is the retention of sodium and water, a process encouraged
by activation of the renin-angiotensin-aldosterone system. In ATN, tubular damage means
that sodium reabsorption is incomplete, and sodium spills out into the urine. There are some
problems with this approach however.

bilateral renal artery stenosis (whole kidney ischemia) or severe glomerulonephritis


(glomerular ischemia) can lead to an artificially low renal sodium even in normovolemia i.e.
the kidneys behave as if they are pre-renally impaired

diuretics can lead to inappropriately high urinary sodium despite volume depletion

many patients have intermediate urinary sodium levels (20-40 mmol/L)

One way around the limitations of urinary sodium is to calculate the fractional excretion of
sodium (FENa). This is the quantity of sodium excreted divided by the quantity filtered, and
expressed as a percentage. The equation for this calculation is (UNa x PCr) / (PNa x UCr) with
the result multiplied by 100 to give the FENa as percentage. In hypovolemia, sodium
reabsorption is appropriately enhanced and the FENa is < 1%. When tubular injury occurs
(ATN) the FENa rises to > 2%.

Obstructive jaundice and hypovolemia do indeed place this man at high risk of acute kidney
injury. In severe pancreatitis there is extensive third space fluid loss, and patients often
require large volumes of intravenous fluid replacement to maintain their cardiac preload.
These should be administered to clinical euvolemia, at least initially, using end-points such as
BP (including postural change), heart rate, and urine output. Central venous pressure
measurement can be useful in some patients, but it is the trend in the CVP and the response
to fluid that is important, not just the number. The controversy over whether crystalloids or
colloids should be used is still ongoing.

Proponents of colloids point to the inevitable redistribution of crystalloid to the extravascular


space after a very short time, leading to very significant edema both visible in the peripheries
and flanks, and invisible in internal organs. Crystalloid proponents point to the lack of studies
demonstrating better outcome, and to the additional expense of colloid. Regardless of the
fluid used, clinical endpoints must be decided upon, targeted, and frequently reassessed, as
should renal function (creatinine).

The administration of furosemide to a hypovolemic oliguric patient, simply to increase their


urine output, is without physiological basis or clinical justification. There are a number of
events during acute pancreatitis that contribute to renal impairment, and none is corrected
by furosemide.

Hypovolemia due to third space fluid loss

peripheral vasodilatation due to the systemic inflammatory response syndrome (SIRS) - this is
the reason for the early rise in CRP and WBC in pancreatitis, not necessarily infection
(although this should of course be excluded)

increased intra-abdominal pressure - it is critical that this not be forgotten. Intra-abdominal


hypertension or the more advanced intra-abdominal compartment syndrome result in renal
venous congestion, potential ureteric obstruction, and a reduction in renal perfusion pressure.
If this is not relieved (through drainage of free intra-abdominal fluid or decompressive
laparotomy) multiple organ failure will follow as gut and renal perfusion continue to fall.
Intra-abdominal pressure can be measured using a pressure transducer attached to the
sampling port on the urinary catheter, which is then occluded after 25 ml of normal saline is
injected in to the bladder.

A 50 year old man presents to your clinic with symptoms of fatigue, arthritis, weight loss and
pain in his lower abdominal area that has been ongoing for the past 6 weeks. The rest of his
history and physical examination were non-specific. Based on your clinical suspicion, you
ordered some screening blood work which showed a high serum iron and ferritin level with
an overall elevated transferrin saturation. As you were suspecting hemochromatosis, further
work-up revealed a real-time PCR showing that the patient is negative for hepatitis C RNA in
his blood serum. Liver biopsy was also ordered that showed no fibrosis or cirrhosis of the liver,
however, iron content was considerably high.

The patients blood (5ml) was sent to the hospital's clinical genetics laboratory to determine
the presence/absence of mutations in the HFE gene. Results from PCR confirm the
inheritance of a susceptibility genotype known to confer a strong risk for developing
hereditary hemochromatosis. The patient was subsequently diagnosed as suffering from
hereditary hemochromatosis and a course of phlebotomy treatment was recommended to
treat and manage the disease.

Which homozygous genotype is responsible for hereditary hemochromatosis in this patient?

a) F508/F508 homozygous
b) C282Y/C282Y homozygous
c) PiZ/PiZ homozygous
d) HbS/HbS homozygous
e) R408W/ R408W homozygous

The correct answer is choice B

Hereditary hemochromatosis is a common autosomal recessive genetic disorder in


Caucasians characterized by severe iron overload. Clinical symptoms are diverse and include
fatigue, arthritis, hepatic cirrhosis, hypogonadism, cardiac arrhythmias, and hypermelanomic
pigmentation of the skin. Iron overload can be fatal as it may lead to organ damage if left
untreated. For most patients, treating this disease involves depletion of iron stores by means
of phlebotomy on a regular weekly basis until symptoms iron stores are reduced to safe levels.
Treament is then limited to a small number of phlebotomy sessions a year to manage iron
stores. Symptoms usually occur in middle-aged adults and exhibit a gender bias, more
commonly affecting men. The molecular mechanisms contributing to hereditary
haemochromatosis involve alterations to proteins crucial to iron homeostasis. The HFE gene
encodes a protein which plays a vital role in the maintenace of iron uptake by cells. Mutations
in this gene can lead to altered HFE expressed and disruption to protein structure,
culminating in increased iron uptake within cells.

Disease association studies have identified a number of mutations in the HFE gene which
have been found to segreagate with hemochromatosis. A point mutation G→A resulting in a
substitution of cysteine for tyrosine at amino acid 282 (designated C282Y) represents the
most potent risk factor in develping heriditary haemocrhomatosis. Homozygosity for this
mutation represents the greatest genetic risk factor for developing the disease (choice B).
Another HFE mutation often found in haemochromatosis patients is know as H63D, a point
mutation C→G resulting in an aspartic acid to histidine substitution. H63D homozygotes have
a milder disease phenotype than C282Y homozygotes. However, in a heterozygous form
alongside C282Y, this allele is known to contribute to iron overload significantly. S65C is
another genetic change at the HFE locus which is associated with hereditary
haemochromatosis and similar to H63D, it gives rise to a milder disease phenotype than
C282Y.

F508 is a common allele in caucasians which is associated with cystic fibrosis (choice a).

PiZ is an allele associated with a severe form of Alpha 1-antitrypsin deficiency (choice c).

HbS is a common causative genetic variant in sickle cell disease (choice d).

A 62 year old man with advanced cirrhosis is treated with lactulose for hepatic
encephalopathy. On physical examination he is confused and has asterixis. Blood pressure is
100/60 while the patient is supine and his pulse rate is 110 beats per minute. There is no
peripheral edema, but ascites is detected. Serum sodium concentration is 160 mmol/L and
potassium is 2.6 mmol/L. The body weight is 64 kg.

This patient's hypernatremia can be best described by which of the following?

a) Excessive insensible losses, primarily of water (euvolemic hypernatremia)

b)Hypotonic losses of sodium and potassium and water (hypovolemic hypernatremia)


c)Hypervolemic hypernatremia

d)Not enough information to determine status

e) None of the above

The correct answer is choice B.

There are a number of key points to take on board:

- hypernatremia is a disorder of water balance, most often a water deficit

- sodium is an effective osmole, so hypernatremia = hyperosmolality

- hyperosmolality leads to cellular dehydration, most often in the brain, and this is serious

- the speed with which hypernatremia occurs determines the symptomatology and the
required rate of collection

- hypernatremia is associated with a higher risk of death in most patient groups

Hypernatremia can occur through loss of water alone without any deficit in sodium (pure
water loss) or can occur in a setting where sodium and other electrolytes are also lost with
water, but the fluid is hypotonic and the water loss greater than the electrolyte loss. These
two mechanisms are the commonest contributors to hypernatremia. It is possible to develop
hypernatremia through administration of excessive sodium e.g. hypertonic saline or
bicarbonate, but this is not common.

Our patient in this case has been taking lactulose as part of the management of his cirrhosis
and encephalopathy, and his case raises a number of issues relating to the genesis and
maintenance of his hypernatremia.

- lactulose causes an osmotic diarrhea in which water is lost in excess of electrolytes


(hypotonic fluid losses)

- encephalopathy can impair our primary defense against hypernatremia, which is thirst and
water ingestion. Elderly, debilitated, or encephalopathic patients are at particular risk

- secondary hyperaldosteronism in severe cirrhosis creates a tendency to sodium retention


and potassium loss

- significant hypokalemia such as this man has may impair renal concentrating mechanisms
preventing proper defense against hypovolemia

Our patient has clinical evidence of hypovolemia, and nothing to suggest excessive insensitive
losses. As the hypernatremia progresses it may worsen the patients CNS status and even
further diminish his chances of self-correcting....so it is up to his doctor to recognize and
correct this abnormality.

At this point you need to be able to work out:


- the size of the water deficit that you need to correct

- the time over which you will correct it

- what effect the administration of different crystalloids will have on serum sodium

There isn't space here to go into that in depth, so please review the references. However,
there are three equations that you will need to keep in mind.

First of all, let's calculate the water deficit:

Water deficit to correct hypernatremia = total body water (TBW, in litres) x [(actual
sodium/desired sodium)-1] and where TBW = 0.6 x lean body weight for men and 0.5 x lean
body weight for women.

e.g. Male with LBW 70kg, Na+ 162: deficit = 0.6 x 70 x [(162/140)-1] = 42 x (1.16 - 1) = 6.7
Litres deficit

Secondly, how quickly should it be corrected?

In general, symptomatic patients who have developed their hypernatremia rapidly over less
than 48 hours should initially have their sodium reduced by 1-1.5 mmol/L/hour until the
sodium is around 150 mmol/L or their symptoms improve. To calculate the deficit to replace
to get down to 150 mmol/L you can insert it into the equation above. If your patient
developed hypernatremia more slowly, then correction should proceed at the slower rate of
0.5 mmol/L/hour because there may have been some osmotic equilibration in the meantime
which places them at greater risk of cerebral edema from over-rapid correction, and this can
have serious consequences including patient death. The equations and suggestions given
here do not constitute medical advice and are for educational purposes only.

Lastly, what are the effects of different fluids on plasma sodium?

This can be calculated from the equations below.

Change in serum Na+ from 1L of infusate = (infusate Na+ - serum Na+)/(TBW +1).

If you fluid contains sodium and potassium then this alters the equation and the volume
required to correct the deficit. The equation becomes:

Change in serum Na+ from 1L of infusate = (infusate Na+ + infusate K+ - serum Na+)/(TBW+1)

Obviously the lower the electrolyte concentration infused the greater the reduction in serum
Na+.
You are called to consult on a 42 year-old man prior to vasectomy the next day. He has a
history of bleeding the day after a wisdom tooth extraction that was performed a few weeks
earlier.

Hgb is 13.4 g/dl (normal 13.2-16.2), HCT 41.6 percent (normal 40-52), WBC is 7,100 cells/ul
(normal 4,500-10,000), platelet count is 350,000/uL (normal 140,000-450,000, bleeding time
is increased to at 20 minutes (normal is 9.5 or less), PT and PTT both are normal, and vWF is
low.

Which of the following is APPROPRIATE?

a) transfusion of platelets
b) administration of factor IX
c) administration of vitamin K
d) administration of hydroxyurea
e) administration of desmopressin

The correct answer is choice E.

A history of moderate bleeding following a dental procedure with no indication of frequent


bleeding throughout life is suggestive of von Willebrand disease, which often is mild or even
asymptomatic, as opposed to hemophilia. Affecting an estimated one percent of the
population, von Willebrand disease is the most common hereditary inherited bleeding
disorder. It results from a deficiency or abnormality of von Willebrand factor (vWF), a
glycoprotein secreted by capillary endothelial cells that acts as a carrier for clotting factor VIII
(FVIII). This protects FVIII from degradation; thus, if vWF is abnormal, the amount of FVIII is
low and the patient's coagulation ability is reduced. Usually symptoms are fairly mild; in many
cases bleeding is a problem only in cases of surgery, or in the case of women during
menstruation.

Assays for vWF antigen test for the presence and concentration of vWF in the blood. vWF is
the antigen that is detected by antibodies using various methods. While this patient's low
vWF indicates von Willebrand disease (specifically vWD type 1, a quantitative deficiency) a
normal vWF antigen test would not be enough to rule out the disease. This is because
abnormally functioning vWF may bind the antibodies used in the assay just as well normal
vWF binds them. For this reason, a von Willebrand factor activity test is performed. Also
known as a ristocetin cofactor test, this assay indicates how well a patient's vWF actually
functions. If a patient has a normal concentration vWF antigen, but an abnormal vWF activity
test, this indicates von Willebrand disease type 2, a functional deficit.

Generally, people with mild von Willebrand disease are treated with desmopressin (choice E),
derivative of the hormone arginine vasopressin, which stimulates capillary endothelial cells to
secrete of vWF. Since the effects of this drug are swift, it is an appropriate treatment for a
patient who is about to give birth. Transfusion of cryoprecipitate also would be useful, since
such preparations contain vWF, along with FVIII. However, transfusion of whole blood (choice
C) is not necessary, since whole blood contains many products, especially red blood cells
(RBCs) that the patient does not need, as she is not anemic.

Since this man's platelet level is normal and even close to the high end of the normal range,
transfusing platelets (choice A) would be of little help. A deficiency in FVIII itself is what is
involved in hemophilia A, which hemophilia B is a deficiency in factor IX (FIX, choice B) and
thus is treated with it.

Vitamin K (choice C) actually is a group of factors that are required for the synthesis of various
proteins, most of which are involved in coagulation. It also plays a role in bone growth and
bone density maintenance. Thus symptoms and signs of deficiency manifest as bleeding
disorders and osteoporosis. Additionally, low levels of vitamin K2 (menaquinone) are
associated with coronary artery disease. Since vitamin K is required for synthesis of clotting
factor VII, deficiency of vitamin K results in an increased prothrombin time (PT). Since this
patient's PT is normal, he does not have a vitamin K deficiency.

Hydroxyurea (choice D) is a treatment given for sickle cell anemia, thus is included purely as a
distraction.

A 64 year old femal, involved in a motor vehicle accident, is brought to the emergency room.
On arrival the patient is breathing spontaneously, non cyanotic, and has no signs of external
trauma. Her vital signs are stable and within the normal range. On examination, she has
bilateral round and reactive pupils of 4mm and patient makes some incomprehensible sounds.
Patient responds to her name by opening her eyes and on applying supraorbital pressure. she
extends her left extremity and grasps your hand with her right hand.

What is her Glasgow coma scale (GCS)?

A) 3

B) 8
C) 10
D) 12
E) 15

The correct answer is choice C.


A 63-year-old alcoholic man presents to the Emergency Department with increasing
abdominal girth and right upper quadrant pain. He has a history of admissions for congestive
heart failure exacerbations, for which he takes diuretics and digitalis. He also has a history of
G6PD and was taking gentamicin a week ago. On physical exam, the patient has a narrow
pulse pressure and an S3 and S4 are heard on auscultation of the heart. Examination of the
right neck veins reveals venous pulsations that are best seen when the head of the bed is
elevated to ninety degrees. The abdomen is distended with shifting dullness.

Which of the following conditions is NOT associated with hypomagnesemia?

a) Alcoholism
b) Antibiotics
c) Digitalis
d) Diuretics
e) Hemolysis

The correct answer is choice E.

A 75 year-old female has been under treatment for arthritis with four weeks of non- steroidal
anti-inflammatory drugs. She now notes two days of increasing chest pressure with exertion
and black, tarry stools. On examination she appears pale, and her vital signs are: Pulse
110/min; BP: 90/60; Resp: 22; Temp: 98.6 F.

Pathophysiologic mechanisms for her accelerated angina pectoris might include all of the
following, EXCEPT:

a) Increased myocardial oxygen demand due to sinus tachycardia


b) Increased left ventricular wall tension due to increased preload
c) Increased myocardial contractility due to increased sympathetic tone
d) Decreased coronary perfusion pressure due to hypotension
e) Decreased oxygen carrying capacity due to anemia

The correct answer is choice B.

Myocardial ischemia is defined by insufficient oxygen delivery to meet the metabolic


demands of the myocardium. Disease processes associated with myocardial ischemia can
therefore be divided into two overlapping etiologic categories: those that cause increased
myocardial oxygen consumption and those that cause decreased oxygen delivery.

The clinical vignette describes a patient with worsening angina in the context of symptoms
and signs suggestive of an upper GI bleed associated with NSAID use. GI bleeds are associated
with loss of intravascular volume as well as well as the osmotically active blood components
that could otherwise cause compensatory fluid shifts from the extravascular space to the
intravascular space. Therefore hemodynamically significant GI bleeds would typically cause
decreased rather than increased preload.

Blood pressure is determined by the product of cardiac output and peripheral resistance. In
order to maintain blood pressure in the setting of a hemodynamically significant loss of
intravsacular volume, the cardiovascular system can compensate by increasing peripheral
resistance and by increasing cardiac output. Cardiac output, in turn, is determined by the
product of the stroke volume and pulse rate. Myocardial oxygen consumption increases in
the setting of compensatory mechanisms to increase cardiac output by increasing pulse rate
(Choice A) and increasing stroke volume (Choice C).

The rate of oxygen delivery supplied by the coronary arteries to the myocardium is dictated
by the rate of blood flow and the oxygen content of the blood. As described in the Poiseuille
equation, blood flow within an artery will vary directly with the magnitude of the pressure
gradient within the artery. The high end of this pressure gradient is determined by the blood
pressure in the aortic root, which is decreased in the context of systemic hypotension (Choice
D).

Although some of the oxygen contained in blood is in solution, blood's oxygen carrying
capacity is driven largely by its hemoglobin content, which is reduced following
hemodynamically significant GI bleeds (Choice E).

A 9 year old girl presents in your clinic with fever, weakness, and aching bones. Two months
earlier, she suffered similar flu-like symptoms including a loss of appetite. These symptoms
subsided after a week but returned a month ago. Since then, she has lost ten percent of her
body weight and now presents with petechiae, and splenomegaly. Her platelet count that is
severely low at 8,000/ul, compared to the normal range of 150,000 to 450,000 platelets/ul.
Her hemoglobin is low at 7.4 grams per deciliter (gm/dl) compared to normal range in
children of 11 to 16 g/dl. Total leukocyte count is 36,000/ul, which is high compared to the
normal range of 4,500-10,000.

Most likely, this girl WILL BE DIAGNOSED WITH which of the following conditions:

a) acute myelogenous leukemia


b) acute lymphoblastic leukemia
c) chronic lymphocytic leukemia
d) chronic myelogenous leukemia
e) Fanconi anemia

The correct answer is choice B.

Presenting symptoms and signs of acute lymphoblastic leukemia (ALL) result from infiltration
of leukemic blast cells
normal, and vWF is low through the body and in the bone marrow, where they interfere with
the development of cells, such as red blood cells (RBCs) and platelets. Interference with RBC
production causes anemia, resulting in weakness, while petechiae (small purple or red spots
on the skin) are due to the low platelet count, thrombocytopenia. Bone pain can result from
the high number of lymphoblasts in bone marrow. Splenomegaly is caused by infiltration of
leukemic blasts in the spleen.

Petechiae are small purple or red spots on the skin that appear frequently when a patient has
a low number of circulating platelets. A normal platelet count in adults is 150,000 to 450,000
platelets per microliter. The risk of bleeding is high when the platelet count falls below 20,000
and becomes severe when the count falls below 10,000.
Although acute myelogenous leukemia (AML) (choice A) can occur through all age groups,
and accounts for 35 percent of childhood leukemia deaths, in school age children, ALL occurs
with four times the incidence of AML.

Chronic lymphocytic leukemia (CLL) (choice C) tends to strike the elderly, or at least people
above age 55 years.
Chronic myelogenous leukemia (CML) (choice D) generally strikes people in their 30s and 40s,
although a highly aggressive form may strike people in their 20s.

Fanconi anemia (choice E) strikes people of a wide range of ages, the mean age being 7 years.
In most cases it is an autosomal recessive disease. Patients may exhibit thrombocytopenia, a
low platelet count, and thus petechiae (small purple or red spots on the skin) like the girl in
this case. As the name implies, they are anemic and thus weak, and in 10 percent of cases
may have leukemia as well. However, they also have a history of certain birth defects, such as
radial ray anomalies and genitourinary abnormalities, along with slow growth, leading to
short stature. Additionally, they often have café au lait spots. areas of skin pigmentation that
vary from light brown to dark brown, akin to coffee mixed with milk.

You might also like